PT31.S3.Q21 - so-called "engineering foods"

lsatseveruslsatseverus Member
edited September 2016 in Logical Reasoning 24 karma
The conclusion here is that: Athletes who need to improve muscle strength should not use the engineered food.

Premises: Hormones produce growth in connective tissues rather than in muscle mass, which does not improve muscular strength

So, I was stuck between answer choices "A" and "C"

I tried the negation test and thought without both of them the argument falls apart.

I chose "A" because "C" was too strong and thought it was an sufficient assumption.

I'm having a hard time eliminating "A" because, I'm assuming if muscle mass does not increase strength, the premises of the argument falls apart and thus breaks the conclusion.
https://7sage.com/lsat_explanations/lsat-31-section-3-question-21/

Comments

  • Rachel YoonRachel Yoon Member
    edited September 2016 173 karma
    This is what I thought when I did this question.

    "Hormones produce growth in connective tissues rather than in muscle mass;this does not improve muscle strength"
    With this part, when we negate the answer choice A, the argument can still stand.
    It can still say, "Yeah, increase in the muscle mass does not produce the effect but growth in connective issue ALSO does not produce the effect. So athletes who need to improve muscle strength should not use the engineered food."

    However, with the answer choice C, if there IS other advantage, then why not use the engineered food? Even if it doesn't produce improved muscle strength, it can still be useful for something else. So "should not use the engineered food" is too much to conclude.

    Hope this can help!
Sign In or Register to comment.